免費論壇 繁體 | 簡體
Sclub交友聊天~加入聊天室當版主
分享
返回列表 发帖
网址:https://www.zhihu.com/question/481037029
标题:以下题目有没有简便的方法?
v2-0aa3e2ea33ab5560f9c9b218be5c2396_r.jpg
2021-8-20 15:36

我的回答:
记 f(x) 的两根为 p, q,则 p+q=-1, pq=2,有
\begin{align*} f(w)f(w^2)f(w^3)f(w^4)={}&(w-p)(w^2-p)(w^3-p)(w^4-p)\\ &\times (w-q)(w^2-q)(w^3-q)(w^4-q), \end{align*}因为 `x^5=1` 的五个根为 `1,w,w^2,w^3,w^4` ,所以
\[x^5-1=(x-1)(x-w)(x-w^2)(x-w^3)(x-w^4),\]代入 p, q 后相乘即得
\[f(w)f(w^2)f(w^3)f(w^4)=\frac {p^5-1}{p-1}\cdot \frac {q^5-1}{q-1}=\frac {(pq)^5-p^5-q^5+1}{pq-p-q+1},\]由 p+q=-1, pq=2 得 `p^5+q^5=(p+q)^5-5pq(p+q)^3+5p^2q^2(p+q)=-11` ,所以
\[f(w)f(w^2)f(w^3)f(w^4)=\frac {2^5+11+1}{2+1+1}=11.\]
发布于 08-20

TOP

网址:https://www.zhihu.com/question/484757364
标题:中科大2021本科开学考试一个等式如何证明?
v2-18785021e1289592c316d68a24ba74ca_r.jpg
2021-9-5 15:13

应该默认了an元素不重复
群友发的题,他说是他们开学考试题,他也不会我就问问。。。
k=3的情况硬算是对的。。。

我的回答:
结论是:奇数时为 1,偶数时为 0。

由于懒得写和式与积式,只写三元和四元,多元是同理的。

三元:对互不相同的 a, b, c 及次数 < 3 的多项式 f(x),由拉格朗**值公式,f(x) 可以写成
\[f(x)=\frac {(x-b)(x-c)}{(a-b)(a-c)}f(a)+\frac {(x-a)(x-c)}{(b-a)(b-c)}f(b)+\frac {(x-a)(x-b)}{(c-a)(c-b)}f(c),\]当 `f(x)=x^2` 时,比较两边的 2 次项系数得
\[1=\frac {a^2}{(a-b)(a-c)}+\frac {b^2}{(b-a)(b-c)}+\frac {c^2}{(c-a)(c-b)},\]当 `f(x)=-x` 时,比较 1 次项系数得
\[-1=\frac {a(b+c)}{(a-b)(a-c)}+\frac {b(a+c)}{(b-a)(b-c)}+\frac {c(a+b)}{(c-a)(c-b)},\]当 `f(x)=1` 时,比较常数项得
\[1=\frac {bc}{(a-b)(a-c)}+\frac {ac}{(b-a)(b-c)}+\frac {ab}{(c-a)(c-b)},\]以上三式相加即得
\[1=\frac {(a+b)(a+c)}{(a-b)(a-c)}+\frac {(b+a)(b+c)}{(b-a)(b-c)}+\frac {(c+a)(c+b)}{(c-a)(c-b)}.\]
四元:对…… a, b, c, d 及次数 < 4 的 f(x) ……写成
\[f(x)=\frac {(x-b)(x-c)(x-d)}{(a-b)(a-c)(a-d)}f(a)+\cdots +\frac {(x-a)(x-b)(x-c)}{(d-a)(d-b)(d-c)}f(d),\]分别令 f(x) 为 `x^3,-x^2,x,-1` 比较系数得
\begin{align*}
1&=\frac {a^3}{(a-b)(a-c)(a-d)}+\cdots +\frac {d^3}{(d-a)(d-b)(d-c)},\\
-1&=\frac {a^2(b+c+d)}{(a-b)(a-c)(a-d)}+\cdots +\frac {d^2(a+b+c)}{(d-a)(d-b)(d-c)},\\
1&=\frac {a(bc+bd+cd)}{(a-b)(a-c)(a-d)}+\cdots +\frac {d(ab+ac+bc)}{(d-a)(d-b)(d-c)},\\
-1&=\frac {bcd}{(a-b)(a-c)(a-d)}+\cdots +\frac {abc}{(d-a)(d-b)(d-c)},
\end{align*}全部相加得
\[0=\frac {(a+b)(a+c)(a+d)}{(a-b)(a-c)(a-d)}+\cdots +\frac {(d+a)(d+b)(d+c)}{(d-a)(d-b)(d-c)}.\]
至此,规律已经很明显,无需再写多元的情况了吧?

编辑于 09-05 01:50

注:链接中的其他解答也用拉格朗,都直接构造出整个一步到位,比我的简洁多了,可以看看。

虽然我的麻烦些,但在证出原等式之余还得到一堆等式,也有点好处吧。

注2:
Nemo
。。。这个关键词屏蔽未免太草

白色闪电
怎么连拉格朗日插值都给屏蔽了啊

kuing (作者) 回复 白色闪电
啊,这是我手工打的星号,自我审查……

TOP

网址:https://www.zhihu.com/question/485052875
标题:请问各位大佬这个应该怎么证明?
v2-32561f021fbf7b79c743a39f71762108_r.jpg
2021-9-8 15:47

我的回答:
令 `\sin x=\sqrt t,0<t<1/2` ,则 `\cos x=\sqrt {1-t}` ,则原不等式取对数等价于
\[\sqrt t\ln \sqrt t<\sqrt {1-t}\ln \sqrt {1-t},\]即 `\sqrt t\ln t<\sqrt {1-t}\ln (1-t)` ,令
\[f(t)=\sqrt t\ln t-\sqrt {1-t}\ln (1-t),t\in \left( {0,\frac 12} \right),\]求二阶导数可得
\[f''(t)=\frac 14\left( {\frac {\ln (1-t)}{(1-t)^{3/2}}-\frac {\ln t}{t^{3/2}}} \right),\]易证 `\frac {\ln x}{x^{3/2}}` 在 (0,1) 上递增,故由 1-t>t 可知 `f''(t)>0` ,从而
\[f(t)<\max \left\{ {f(0^+),f\left( {\frac 12} \right)} \right\}=0,\]即得证。

发布于 09-08

TOP

网址:https://www.zhihu.com/question/502137064
标题:如何证明任何关于cosπ/7或cosπ/9的有理函数一定是三次方程的根?
设任意有理函数
\[f(t)= \frac{a_0+a_1t+a_2t^2+\cdots +a_mt^m}{b_0+b_1t+b_2t^2+\cdots +b_nt^n},\]其中 $a_i,b_j\left( i=0,1,2,\cdots ,m,j=0,1,2,\cdots ,n \right)  $ 都是整数.
那么 $f\left( \cos \frac{\pi}{7} \right)  $ 和 $ f\left( \cos \frac{\pi}{9} \right)  $ 一定是一个整系数三次方程的根.

比如 $\frac{1+\cos \frac{\pi}{7}+\cos ^2\frac{\pi}{7}}{1+\cos \frac{\pi}{7}+\cos ^3\frac{\pi}{7}} $ 是方程 $223 x^3-1732 x^2+2860 x-1352=0$ 的根.

如果反过来,是否所有整系数三次方程的根都可以用关于$\cos \frac{\pi}{k}(k \in  \mathbb{N} _+)  $ 的有理函数表示? 如果不能,那什么样的三次方程,它的根才可以用关于三角函数的有理函数表示?

这个问题是从Vasile不等式的取等条件想到的.

我的回答:
记 $t=\cos \frac \pi 7$ ,由 $\cos \frac {3\pi }7+\cos \frac {4\pi }7=0$ 可知 t 满足 $8t^3-4t^2-4t+1=0$ 。

设 f(t) = u,分子分母分别写成
\begin{align*}
a_0+a_1t+a_2t^2+\cdots +a_mt^m&=q_1(t)(8t^3-4t^2-4t+1)+r_1(t),\\
b_0+b_1t+b_2t^2+\cdots +b_nt^n&=q_2(t)(8t^3-4t^2-4t+1)+r_2(t),
\end{align*}其中 $\deg (r_1(t)),\deg (r_2(t))<3$ 且系数均为有理数,于是 f(t) = u 变成 $\frac {r_1(t)}{r_2(t)}=u$ ,去分母化为 $c_0(u)+c_1(u)t+c_2(u)t^2=0$ ,其中 $c_i(u)$ 均为关于 u 的一次函数或常数,且系数均为整数,于是有方程组
\[
\left\{\begin{aligned}
8t^3-4t^2-4t+1&=0,\\
c_0(u)+c_1(u)t+c_2(u)t^2&=0,
\end{aligned}\right.
\]用结式消去 t,即
\[
\begin{vmatrix}  8 & -4 & -4 & 1 & 0 \\  0 & 8 & -4 & -4 & 1 \\  c_2(u) & c_1(u) & c_0(u) & 0 & 0 \\  0 & c_2(u) & c_1(u) & c_0(u) & 0 \\  0 & 0 & c_2(u) & c_1(u) & c_0(u) \end{vmatrix}=0,
\]由于 $c_i(u)$ 不超过一次,且系数均为整数,所以上式展开后关于 u 的次数不超过三次,系数亦均为整数。

cosπ/9 的同理可证。

最后的问题不会……

编辑于 2021-11-28 15:56

注:上述证明具有一般性,即:设 `t` 是 `k` 次整系数方程的根,`f(x)` 为有理函数,则 `f(t)` 是不超过 `k` 次整系数方程的根。

TOP

网址:https://www.zhihu.com/question/503305558
标题:怎么求解这种4元方程组比较简便?
v2-cbe6d9f752a53faaf2d2105fd2b75ced_r.jpg
2021-12-5 14:29

我的回答:令 $a_n=A_0x_0^n+A_1x_1^n$ ,则存在 p, q 使 $a_{n+2}=pa_{n+1}+qa_n$ ,所以有 2=p+q, 6=2p+q,得 p=4, q=-2,即 $a_{n+2}=4a_{n+1}-2a_n$ ,其特征方程 $x^2=4x-2$ 解得 $x=2\pm \sqrt 2$ ,所以 $x_0=2\pm \sqrt 2,x_1=2\mp \sqrt 2$ ,再由 $A_0+A_1=1,A_0x_0+A_1x_1=1$ 算出 $A_0,A_1$ ,最终结果为
\[x_0=2\pm \sqrt 2,x_1=2\mp \sqrt 2,A_0=\frac {2\mp \sqrt 2}4,A_1=\frac {2\pm \sqrt 2}4.\]
发布于 2021-12-03 15:44

TOP

结论是:奇数时为 1,偶数时为 0。

由于懒得写和式与积式,只写三元和四元,多元是同理的。

三元:对互 ...
kuing 发表于 2021-9-5 15:13


我又試吓解哩個
Summation with polynomial roots

$\displaystyle S_{n,m}=\sum_{i=1}^m x_i^n \prod_{j=1\atop i\neq j}^m\frac{1}{x_i-x_j}
=\begin{cases}0 & 0\le n\le m-2\\1 & n=m-1\end{cases}$

$a_{k+1}=0$
$\displaystyle \sum_{i=1}^k \frac{1}{a_i}\prod_{j=1\atop j\neq i}^k \frac{1}{a_i-a_j}
=\sum_{i=1}^k \prod_{j=1\atop j\neq i}^{k+1} \frac{1}{a_i-a_j}
=-\prod_{j=1}^k \frac{1}{0-a_j}+\sum_{i=1}^{k+1} \prod_{j=1\atop j\neq i}^{k+1} \frac{1}{a_i-a_j}$
$\displaystyle =(-1)^{k-1}\prod_{j=1}^k \frac{1}{a_j}$

$\displaystyle \sum_{i=1}^k \prod_{j=1\atop j\neq i}^k \frac{a_i+a_j}{a_i-a_j}
=\sum_{i=1}^k \frac{1}{2a_i}\prod_{j=1}^k (a_i+a_j)\prod_{j=1\atop j\neq i}^k \frac{1}{a_i-a_j}$
$\displaystyle =\sum_{i=1}^k \frac{1}{2a_i}(a_i^k+a_i^{k-1}\sum_{j=1}^k a_j+\cdots+\prod_{j=1}^k a_j)\prod_{j=1\atop j\neq i}^k \frac{1}{a_i-a_j}$
$\displaystyle =\frac{1+(-1)^{k-1}}{2}$
1

评分人数

TOP

新年第一撸,竟然是道高数题……[流汗黄豆表情]
网址:https://www.zhihu.com/question/508910626
标题:这个极限怎么做啊呢?
\[\lim_{x\to 0} \frac {(1+\sin x)^\frac 1{\sin x}-(1+\tan x)^\frac 1{\tan x}}{x^3}.\]

我的回答:
令 `f(x)=(1+x)^{1/x}`,求导得
\[f'(x)=(1+x)^{1/x}\frac{x-(1+x)\ln(1+x)}{x^2(1+x)},\]

\[\lim_{x\to0}f'(x)=e\lim_{x\to0}\frac{x-(1+x)\ln(1+x)}{x^2}=e\lim_{x\to0}\frac{-\ln(1+x)}{2x}=-\frac e2,\]
于是对所求极限有
\begin{align*}
\text{原式}&=\lim_{x\to0}\frac{f(\sin x)-f(\tan x)}{\sin x-\tan x}\cdot\frac{\sin x-\tan x}{x^3}\\
&=\lim_{x\to0}f'(x)\cdot\frac{-2\sin x\sin^2\frac x2}{x^3\cos x}\\
&=-\frac e2\cdot\frac{-2}{2^2}\\
&=\frac e4.
\end{align*}

TOP

返回列表 回复 发帖